Zadania z modulo

Podzielność. Reszty z dzielenia. Kongruencje. Systemy pozycyjne. Równania diofantyczne. Liczby pierwsze i względnie pierwsze. NWW i NWD.
Krodinor
Użytkownik
Użytkownik
Posty: 207
Rejestracja: 13 sty 2016, o 00:09
Płeć: Mężczyzna
Lokalizacja: Poznań
Podziękował: 7 razy
Pomógł: 12 razy

Zadania z modulo

Post autor: Krodinor »

1. Rozwiązać kongurencje \(\displaystyle{ 3x\equiv 7 \pmod{11}}\)
Wystarczy pomnożyć razy \(\displaystyle{ 4}\) i dostaniemy, że \(\displaystyle{ x\equiv 6\pmod{11}}\)?

2. Do jakiej reszty przystaje \(\displaystyle{ 3^{10536}\mod{11}}\)?
Tutaj nie mam za bardzo pomysłu.
Ostatnio zmieniony 8 lut 2017, o 23:32 przez Jan Kraszewski, łącznie zmieniany 2 razy.
Powód: Poprawa wiadomości.
szw1710

Zadania z modulo

Post autor: szw1710 »

2. Wskazówka: \(\displaystyle{ 3^5\equiv 1}\)
ajlofmath
Użytkownik
Użytkownik
Posty: 20
Rejestracja: 22 paź 2016, o 14:49
Płeć: Mężczyzna
Lokalizacja: Olszyna
Podziękował: 2 razy

Zadania z modulo

Post autor: ajlofmath »

W pierwszym taktycznym ruchem było by równanie \(\displaystyle{ 3x+4=11y}\)
W drugim dzięki Eulerowi wiemy, że
\(\displaystyle{ 3 ^{10530} \equiv 1\pmod{11}}\)
Więc \(\displaystyle{ 3 ^{10536} \equiv 3 ^{6}\pmod{11}}\)

Można też zauważyć, że \(\displaystyle{ 3 ^{5} \equiv 1\pmod{11}}\)
zatem \(\displaystyle{ 3 ^{10535} \equiv 1\pmod{11}}\)
Ostatnio zmieniony 8 lut 2017, o 23:30 przez Jan Kraszewski, łącznie zmieniany 2 razy.
Powód: Poprawa wiadomości.
Krodinor
Użytkownik
Użytkownik
Posty: 207
Rejestracja: 13 sty 2016, o 00:09
Płeć: Mężczyzna
Lokalizacja: Poznań
Podziękował: 7 razy
Pomógł: 12 razy

Zadania z modulo

Post autor: Krodinor »

A zatem wynik w drugim to będzie \(\displaystyle{ 3\mod 11}\)?

W pierwszym natomiast rozumiem, że trzeba jeszcze zauważyć, że \(\displaystyle{ NWD(3,11)}\) to \(\displaystyle{ 1}\), ale czy poza tym jest dobrze?
Ostatnio zmieniony 8 lut 2017, o 23:31 przez Jan Kraszewski, łącznie zmieniany 1 raz.
Powód: Poprawa wiadomości.
ajlofmath
Użytkownik
Użytkownik
Posty: 20
Rejestracja: 22 paź 2016, o 14:49
Płeć: Mężczyzna
Lokalizacja: Olszyna
Podziękował: 2 razy

Zadania z modulo

Post autor: ajlofmath »

3 i 11 są pierwsze, więc pominąłem to sprawdzenie i tak jest dobrze, bo to podnosisz \(\displaystyle{ 3 ^{10}}\) do potęgi 1053 to samo robisz z jedynką, więc masz jeden.
Krodinor
Użytkownik
Użytkownik
Posty: 207
Rejestracja: 13 sty 2016, o 00:09
Płeć: Mężczyzna
Lokalizacja: Poznań
Podziękował: 7 razy
Pomógł: 12 razy

Zadania z modulo

Post autor: Krodinor »

Czyli ostatecznie rozwiązania to:
1. \(\displaystyle{ x\equiv 6\pmod{11}}\)
2. \(\displaystyle{ 3 \mod 11}\)
Ostatnio zmieniony 8 lut 2017, o 23:31 przez Jan Kraszewski, łącznie zmieniany 1 raz.
Powód: Poprawa wiadomości.
ajlofmath
Użytkownik
Użytkownik
Posty: 20
Rejestracja: 22 paź 2016, o 14:49
Płeć: Mężczyzna
Lokalizacja: Olszyna
Podziękował: 2 razy

Zadania z modulo

Post autor: ajlofmath »

Czy coś ci wskazuje, że może być inaczej.
ODPOWIEDZ